0 Daumen
466 Aufrufe

Aufgabe:

f(x) = 8e2x _ log5(x)


Problem/Ansatz:

… "Bestimmen Sie die Stelle an welcher der Graph von ()

eine waagerechte  

Tangente besitzt. Wenden Sie die Newton-Iteration mit dem Startwert 0 = 1 an.

Bearbeitungshinweis:

• Bestimmen Sie die Stelle auf 4 Nachkommastellen genau."

Das ist Text der Aufgabe, Aber wenn ich mit dem Startwert beginne, habe ich eine negative Zahl für x, und das kann nicht in Log benutzen, wie kann ich diese Problem losen?

Avatar von

1. Meinst Du \( f(x) = 8 e^{2x} - \log_5(x)  \)?

2. Was ist mit Graph von () gemeint?

Was ist mit Wenden Sie die Newton-Iteration mit dem Startwert 0 = 1 an, gemeint?

Ein bisschen mehr Sorgfalt bei der Aufgabenbeschreibung wäre angebracht.

Das ist die rightige Frage, ich habe das falsch geschrieben.

"Bestimmen Sie die Stelle an welcher der Graph von f(x) = 8e2x _ log5(x)

eine waagerechte 

Tangente besitzt. Wenden Sie die Newton-Iteration mit dem Startwert X = 1 an."

2 Antworten

0 Daumen
 
Beste Antwort

Die Newton Iteration sieht so aus:

$$  x_{n+1} = x_n - \frac{ f'(x_n)}{f''(x)}  $$ mit \( f'(x) = 16e^{2x} - \frac{1}{x \ln(5)} \) und

\( f''(x) = 32 e^{2x} +\frac{1}{x^2 \ln(5)} \)

Das Ergebnis ist dann

blob.png

Avatar von 39 k
0 Daumen

f(x) = 8e^(2x) - log5(x) = f(x) = 8e^(2x) - ln(x) / ln(5)

==> f ' (x) =  16e^(2x) - (1 / ln(5)) * 1/x

Hiervon suchst du eine Nullstelle . Brauchst also

  f ' ' (x) = 32e^(2x) +  (1 / ln(5)) * 1/x^2

x0=1 ==>   x1 = 1  -  (  16*e^2 - 1 / ln(5) ) /  (   32e^(2) +  (1 / ln(5))           ) = 1 -  117,604 / 237,071 = 0,5039

Vielleicht kommst du damit weiter.

Da kommst du am Ende auf 0,036127...

Avatar von 288 k 🚀

Ja das ist rightig, aber im nächste Schritt ist x0 = 0.49735834987 und f(x0) / f ' (x0) = 0.52519639845

Also das bedeutet die nächste Wert von x ist : _0.02783804858 und hier kann ich nicht in f(x0) und  f ' (x0) benutzen.

Ich hoffe, dass Sie mir helfen.

Und Danke für Ihre Antwort.

Ich bekomme für den nächsten Wert

0,5039  - f ' (0,5039 ) / f ' ' (0,5039 )

=0,5039  - 42,603 / 90,118

=0,5039  - 0,4727

= 0,0312

und dann

 0,0312  - f ' ( 0,0312 ) / f '' ( 0,0312 )

= 0,0312  - (-2,8924) / 672,84

= 0,0312  + 0,0043

=0,0354

also schon recht wenig Unterschied zum Vorgänger.

Vielen Dank für Ihre Antwort.

Ein anderes Problem?

Stell deine Frage

Willkommen bei der Mathelounge! Stell deine Frage einfach und kostenlos

x
Made by a lovely community